How to prove 'infinite primes' kind of problem

  • Thread starter l-1j-cho
  • Start date
  • Tags
    Primes
In summary: It is just a different way of presenting Euclid's proof. In summary, there are many different proofs of the infinitude of prime numbers, including the Euclidean proof and proofs using prime factorials, Fermat numbers, Euler's zeta-function, and more. However, none of these proofs have been able to prove conjectures such as the twin prime conjecture, Mersenne prime conjecture, or Sophie-Germain conjecture.
  • #1
l-1j-cho
104
0
Other than the Eucledean method
(1+p1p2p3...)and so on

other than this, how do we prove that there are infinite prime numbers?
in algebreic way (excluding analyses)
 
Last edited:
Physics news on Phys.org
  • #2
The Euclidean proof is so neat and simple, I cannot imagine why you would want a different proof.
 
  • #3
indeed

but I was just wondering if there is any other technique to prove there are infinite primes so it can be applied to prove twin prime conjecture, mersenne prime conjecture, or sophie-germain conjecture.
 
  • #4
There are many many proofs of the infinitude of the primes

http://primes.utm.edu/notes/proofs/infinite/

And about 1/4 of the way down through this
http://en.wikipedia.org/wiki/Prime_number

and many more, but I hesitate to claim an infinite number of them.

However thus far nobody no one has seen how to use these to prove any of the conjectures you mention.

On a brighter and prime related note, there is now a non-crank proposed proof of Goldbach that has been submitted. It would be interesting to see if that could be explained in a form that those not in the "bright student" group could understand.
 
  • #5
One proof due to the German mathematician Kummer:

Assumption:
Let P[itex]_{r}[/itex]:={2,3,5,7,...,p[itex]_{r}[/itex]} the finite set of all primes

form the prime factorial p[itex]_{r}[/itex]#:=2*3*5*...*p[itex]_{r}[/itex]
and the number q:=-1+ p[itex]_{r}[/itex]#

q is not in P[itex]_{r}[/itex], that means, q is composite, with prime
factors from P[itex]_{r}[/itex]; let p[itex]_{k}[/itex] be one of those factors

Now: p[itex]_{k}[/itex] divides both p[itex]_{r}[/itex]# and q, hence it
must divide their difference: but this difference is 1
and p[itex]_{k}[/itex] does NOT divide 1.

Therefore our assumption must be false
 
  • #6
The above proof is essentially equivalent to euclids proof.
 
  • #7
disregardthat said:
The above proof is essentially equivalent to euclids proof.

I think not.

Recall the original version of Euclids proof:

Let Q[itex]_{r}[/itex]:={q[itex]_{1}[/itex], q[itex]_{2}[/itex],...,q[itex]_{r}[/itex]} be the set a r distinct
prime numbers, and set q:=1+q[itex]_{1}[/itex]*...*q[itex]_{r}[/itex]

(Note: the r primes must NOT be the first r primes 2,3,..,p[itex]_{r}[/itex] !)

Then q is NOT divisible by any of the q[itex]_{i}[/itex] and must therefore be an new
prime or at least a composite number formed with additional primes.

Therefore: from any Q[itex]_{r}[/itex] you can construct a Q[itex]_{r+1}[/itex], and that proves the infinitude of the primes

Now compare this to the Kummer proof I cited.
 
  • #8
The following proff is due to Polya

Let e:=2[itex]^{k}[/itex] and F[itex]_{k}[/itex]:=1+2[itex]^{e}[/itex] be the Fermat numbers

with F[itex]_{0}[/itex]:=3, F[itex]_{1}[/itex]:=5, F[itex]_{3}[/itex]:=17. F[itex]_{4}[/itex]:=257, F[itex]_{5}[/itex]:=65537, F[itex]_{6}[/itex]:=4294967297, etc

we have the recurrence relation F[itex]_{k+1}[/itex]:=2+[itex]\prod[/itex][itex]^{k}_{i=0}[/itex]F[itex]_{i}[/itex]

From the recurrence relation we have the following

Lemma: Any two F[itex]_{i}[/itex] and F[itex]_{j}[/itex] for distinct i, j are co-prime,
i.e. have no factors in common


Now let F:={F[itex]_{0}[/itex],F[itex]_{1}[/itex],F[itex]_{2}[/itex],...} be the set of all Fermat numbers and this set is countable,
i.e. has the cardinality of N:={1,2,3,4,...}, the set of the natural numbers

Therefore no finite P[itex]_{r}[/itex]:={2,3,5,...,p[itex]_{r}[/itex]} can meet the requirements
of our Lemma, there would simply be not enough primes.
 
  • #9
Let P:={p[itex]_{i}[/itex]} = {2,3,5,7,...} and
E[itex]_{k}[/itex]:=1+[itex]\prod[/itex][itex]^{k}_{i=1}[/itex]p[itex]_{i}[/itex] be the Euclid numbers with E[itex]_{1}[/itex]:=3, E[itex]_{2}[/itex]:=7, E[itex]_{3}[/itex]:=31. etc

We have the following

Lemma All E[itex]_{i}[/itex] are odd, i.e. 2 is not a factor of E[itex]_{i}[/itex].

and it is not difficult to prove the following

Theorem Any two E[itex]_{i}[/itex], E[itex]_{j}[/itex] for different i,j are co-prime, i.e. have no common factors.

Now supose, P:={2,3,5,7,...,p[itex]_{r}[/itex]}, then the corresponding {E[itex]_{i}[/itex]} must be the numbers {2,3,5,7,...,p[itex]_{r}[/itex]}
in some order (our Theorem), but that is impossible (our Lemma)

This proof was inspired by the Polya proof
 
  • #10
From the Euler [itex]\zeta[/itex]-function [itex]\zeta[/itex](s):=[itex]\sum[/itex][itex]^{\infty}_{i=1}[/itex]([itex]\frac{1}{i}[/itex])[itex]^{s}[/itex]

we know, that [itex]\zeta[/itex](2) is [itex]\frac{\pi^{2}}{6}[/itex]

This quantity is irrational, since [itex]\pi[/itex] itself is irrational

From the Euler product formula, we have

[itex]\zeta[/itex](2):=[itex]\prod[/itex](1 - [itex]\frac{1}{p^{2}}[/itex])[itex]^{-1}[/itex], where the product is taken over all primes

If there were only a finite number of primes, this product would
be a rational number, in contradiction to the irrational nature of [itex]\frac{\pi^{2}}{6}[/itex],
which concludes the proof.
 
  • #11
RamaWolf said:
I think not.

Recall the original version of Euclids proof:

Let Q[itex]_{r}[/itex]:={q[itex]_{1}[/itex], q[itex]_{2}[/itex],...,q[itex]_{r}[/itex]} be the set a r distinct
prime numbers, and set q:=1+q[itex]_{1}[/itex]*...*q[itex]_{r}[/itex]

(Note: the r primes must NOT be the first r primes 2,3,..,p[itex]_{r}[/itex] !)

Then q is NOT divisible by any of the q[itex]_{i}[/itex] and must therefore be an new
prime or at least a composite number formed with additional primes.

Therefore: from any Q[itex]_{r}[/itex] you can construct a Q[itex]_{r+1}[/itex], and that proves the infinitude of the primes

Now compare this to the Kummer proof I cited.

Yes, it is not equivalent, but "essentially" equivalent. It doesn't contain any new ideas.
 

1. How can we prove that there are an infinite number of prime numbers?

There are a few different methods that can be used to prove the infinitude of prime numbers. The most common approach is to use a proof by contradiction, where we assume that there are a finite number of primes and then show that this leads to a contradiction. Another method is to use the Fundamental Theorem of Arithmetic, which states that every positive integer can be uniquely expressed as a product of primes. By demonstrating that there is no largest prime number, we can prove that there must be an infinite number of primes.

2. Is there a mathematical formula or algorithm for generating an infinite number of prime numbers?

No, there is no known formula or algorithm that can generate an infinite number of prime numbers. However, there are some methods that can be used to efficiently generate large prime numbers. One example is the Sieve of Eratosthenes, which involves systematically eliminating non-prime numbers to reveal the prime numbers.

3. Can we prove that there are an infinite number of prime numbers using only mathematical reasoning?

Yes, the infinitude of prime numbers can be proven using purely mathematical reasoning and logical arguments. This is known as a proof by deduction, where we use established mathematical principles and axioms to arrive at a conclusion.

4. Are there any real-world applications or implications of the infinitude of prime numbers?

While the concept of an infinite number of primes may seem abstract, it has a number of practical applications in fields such as cryptography and computer science. Prime numbers are used in encryption algorithms to secure sensitive information, and the study of prime numbers has also led to advancements in number theory and other areas of mathematics.

5. Are there any unsolved problems or open questions related to the infinitude of prime numbers?

Yes, there are still many open questions and unsolved problems related to prime numbers. For example, the Twin Prime Conjecture, which states that there are an infinite number of prime pairs that differ by 2, remains unproven. Additionally, the distribution of primes and the existence of a pattern or formula that can generate prime numbers is still a topic of ongoing research and debate.

Similar threads

Replies
8
Views
376
  • Linear and Abstract Algebra
Replies
1
Views
2K
  • Precalculus Mathematics Homework Help
Replies
12
Views
1K
  • Precalculus Mathematics Homework Help
Replies
2
Views
903
  • Linear and Abstract Algebra
Replies
3
Views
796
Replies
2
Views
2K
  • Precalculus Mathematics Homework Help
Replies
9
Views
1K
  • Linear and Abstract Algebra
Replies
2
Views
1K
  • Linear and Abstract Algebra
Replies
11
Views
2K
  • Linear and Abstract Algebra
Replies
4
Views
941
Back
Top